Difference between revisions of "2020 AMC 10A Problems/Problem 23"

(Solution)
(Video Solution)
(12 intermediate revisions by 10 users not shown)
Line 1: Line 1:
 +
{{duplicate|[[2020 AMC 12A Problems|2020 AMC 12A #20]] and [[2020 AMC 10A Problems|2020 AMC 10A #23]]}}
 +
 
== Problem ==
 
== Problem ==
 
Let <math>T</math> be the triangle in the coordinate plane with vertices <math>(0,0), (4,0),</math> and <math>(0,3).</math> Consider the following five isometries (rigid transformations) of the plane: rotations of <math>90^{\circ}, 180^{\circ},</math> and <math>270^{\circ}</math> counterclockwise around the origin, reflection across the <math>x</math>-axis, and reflection across the <math>y</math>-axis. How many of the <math>125</math> sequences of three of these transformations (not necessarily distinct) will return <math>T</math> to its original position? (For example, a <math>180^{\circ}</math> rotation, followed by a reflection across the <math>x</math>-axis, followed by a reflection across the <math>y</math>-axis will return <math>T</math> to its original position, but a <math>90^{\circ}</math> rotation, followed by a reflection across the <math>x</math>-axis, followed by another reflection across the <math>x</math>-axis will not return <math>T</math> to its original position.)
 
Let <math>T</math> be the triangle in the coordinate plane with vertices <math>(0,0), (4,0),</math> and <math>(0,3).</math> Consider the following five isometries (rigid transformations) of the plane: rotations of <math>90^{\circ}, 180^{\circ},</math> and <math>270^{\circ}</math> counterclockwise around the origin, reflection across the <math>x</math>-axis, and reflection across the <math>y</math>-axis. How many of the <math>125</math> sequences of three of these transformations (not necessarily distinct) will return <math>T</math> to its original position? (For example, a <math>180^{\circ}</math> rotation, followed by a reflection across the <math>x</math>-axis, followed by a reflection across the <math>y</math>-axis will return <math>T</math> to its original position, but a <math>90^{\circ}</math> rotation, followed by a reflection across the <math>x</math>-axis, followed by another reflection across the <math>x</math>-axis will not return <math>T</math> to its original position.)
Line 5: Line 7:
  
 
== Solution ==
 
== Solution ==
 +
<asy>
 +
size(10cm);
 +
Label f;
 +
f.p=fontsize(6);
 +
xaxis(-6,6,Ticks(f, 2.0));
 +
yaxis(-6,6,Ticks(f, 2.0));
 +
 +
filldraw(origin--(4,0)--(0,3)--cycle, gray, black+linewidth(1));
 +
</asy>
  
 
First, any combination of motions we can make must reflect <math>T</math> an even number of times. This is because every time we reflect <math>T</math>, it changes orientation. Once <math>T</math> has been flipped once, no combination of rotations will put it back in place because it is the mirror image; however, flipping it again changes it back to the original orientation. Since we are only allowed <math>3</math> transformations and an even number of them must be reflections, we either reflect <math>T</math> <math>0</math> times or <math>2</math> times.
 
First, any combination of motions we can make must reflect <math>T</math> an even number of times. This is because every time we reflect <math>T</math>, it changes orientation. Once <math>T</math> has been flipped once, no combination of rotations will put it back in place because it is the mirror image; however, flipping it again changes it back to the original orientation. Since we are only allowed <math>3</math> transformations and an even number of them must be reflections, we either reflect <math>T</math> <math>0</math> times or <math>2</math> times.
 +
 +
  
 
Case 1: 0 reflections on T
 
Case 1: 0 reflections on T
 +
 +
  
 
In this case, we must use <math>3</math> rotations to return <math>T</math> to its original position. Notice that our set of rotations, <math>\{90^\circ,180^\circ,270^\circ\}</math>, contains every multiple of <math>90^\circ</math> except for <math>0^\circ</math>. We can start with any two rotations <math>a,b</math> in <math>\{90^\circ,180^\circ,270^\circ\}</math> and there must be exactly one <math>c \equiv -a - b \pmod{360^\circ}</math> such that we can use the three rotations <math>(a,b,c)</math> which ensures that <math>a + b + c \equiv 0^\circ \pmod{360^\circ}</math>. That way, the composition of rotations <math>a,b,c</math> yields a full rotation. For example, if <math>a = b = 90^\circ</math>, then <math>c \equiv -90^\circ - 90^\circ = -180^\circ \pmod{360^\circ}</math>, so <math>c = 180^\circ</math> and the rotations <math>(90^\circ,90^\circ,180^\circ)</math> yields a full rotation.
 
In this case, we must use <math>3</math> rotations to return <math>T</math> to its original position. Notice that our set of rotations, <math>\{90^\circ,180^\circ,270^\circ\}</math>, contains every multiple of <math>90^\circ</math> except for <math>0^\circ</math>. We can start with any two rotations <math>a,b</math> in <math>\{90^\circ,180^\circ,270^\circ\}</math> and there must be exactly one <math>c \equiv -a - b \pmod{360^\circ}</math> such that we can use the three rotations <math>(a,b,c)</math> which ensures that <math>a + b + c \equiv 0^\circ \pmod{360^\circ}</math>. That way, the composition of rotations <math>a,b,c</math> yields a full rotation. For example, if <math>a = b = 90^\circ</math>, then <math>c \equiv -90^\circ - 90^\circ = -180^\circ \pmod{360^\circ}</math>, so <math>c = 180^\circ</math> and the rotations <math>(90^\circ,90^\circ,180^\circ)</math> yields a full rotation.
  
The only case in which this fails is when <math>c</math> would have to equal <math>0^\circ</math>. This happens when <math>(a,b)</math> is already a full rotation, namely, <math>(a,b) = (90^\circ,270^\circ),(180^\circ,180^\circ),</math> or <math>(270^\circ,90^\circ)</math>. However, we can simply subtract these three cases from the total. Selecting <math>(a,b)</math> from <math>\{90^\circ,180^\circ,270^\circ\}</math> yeilds <math>3 \cdot 3 = 9</math> choices, and with <math>3</math> that fail, we are left with <math>6</math> combinations for case 1.
+
The only case in which this fails is when <math>c</math> would have to equal <math>0^\circ</math>. This happens when <math>(a,b)</math> is already a full rotation, namely, <math>(a,b) = (90^\circ,270^\circ),(180^\circ,180^\circ),</math> or <math>(270^\circ,90^\circ)</math>. However, we can simply subtract these three cases from the total. Selecting <math>(a,b)</math> from <math>\{90^\circ,180^\circ,270^\circ\}</math> yields <math>3 \cdot 3 = 9</math> choices, and with <math>3</math> that fail, we are left with <math>6</math> combinations for case 1.
 +
 
 +
 
  
 
Case 2: 2 reflections on T
 
Case 2: 2 reflections on T
 +
 +
  
 
In this case, we first eliminate the possibility of having two of the same reflection. Since two reflections across the x-axis maps <math>T</math> back to itself, inserting a rotation before, between, or after these two reflections would change <math>T</math>'s final location, meaning that any combination involving two reflections across the x-axis would not map <math>T</math> back to itself. The same applies to two reflections across the y-axis.
 
In this case, we first eliminate the possibility of having two of the same reflection. Since two reflections across the x-axis maps <math>T</math> back to itself, inserting a rotation before, between, or after these two reflections would change <math>T</math>'s final location, meaning that any combination involving two reflections across the x-axis would not map <math>T</math> back to itself. The same applies to two reflections across the y-axis.
Line 20: Line 39:
 
Therefore, we must use one reflection about the x-axis, one reflection about the y-axis, and one rotation. Since a reflection about the x-axis changes the sign of the y component, a reflection about the y-axis changes the sign of the x component, and a <math>180^\circ</math> rotation changes both signs, these three transformation composed (in any order) will suffice. It is therefore only a question of arranging the three, giving us <math>3! = 6</math> combinations for case 2.
 
Therefore, we must use one reflection about the x-axis, one reflection about the y-axis, and one rotation. Since a reflection about the x-axis changes the sign of the y component, a reflection about the y-axis changes the sign of the x component, and a <math>180^\circ</math> rotation changes both signs, these three transformation composed (in any order) will suffice. It is therefore only a question of arranging the three, giving us <math>3! = 6</math> combinations for case 2.
  
Combining both cases we get <math>6 + 6 = \textbf{(A) } 12</math>
+
Combining both cases we get <math>6 + 6 = \boxed{\textbf{(A) } 12}</math>
 +
 
 +
==Solution 2(Rewording solution 1)==
 +
 
 +
As in the previous solution, note that we must have either 0 or 2 reflections because of orientation since reflection changes orientation that is impossible to fix by rotation. We also know we can't have the same reflection twice, since that would give a net of no change and would require an identity rotation.
 +
 
 +
Suppose there are no reflections. Denote <math>90^{\circ}</math> as 1, <math>180^{\circ}</math> as 2, and <math>270^{\circ}</math> as 3, just for simplification purposes. We want a combination of 3 of these that will sum to either 4 or 8(0 and 12 is impossible since the minimum is 3 and the max is 9). 4 can be achieved with any permutation of <math>(1-1-2)</math> and 8 can be achieved with any permutation of <math>(2-3-3)</math>. This case can be done in <math>3+3=6</math> ways.
 +
 
 +
Suppose there are two reflections. As noted already, they must be different, and as a result will take the triangle to the opposite side of the origin if we don't do any rotation. We have 1 rotation left that we can do though, and the only one that will return to the original position is 2, which is <math>180^{\circ}</math> AKA reflection across origin. Therefore, since all 3 transformations are distinct. The three transformations can be applied anywhere since they are commutative(think quadrants). This gives <math>6</math> ways.
 +
 
 +
<math>6+6=\boxed{(A)12}</math>
 +
 
 +
== Video Solution ==
 +
 
 +
Education, The Study of Everything
 +
 
 +
https://youtu.be/SBhkM2frTUA
 +
 
 +
https://youtu.be/5TjrDCxTm7Q - Richard Rusczyk
 +
 
 +
== Video Solution ==
 +
 
 +
https://www.youtube.com/watch?v=iXwvTmFvo0c ~ MathEx
  
 
==See Also==
 
==See Also==
  
 
{{AMC10 box|year=2020|ab=A|num-b=22|num-a=24}}
 
{{AMC10 box|year=2020|ab=A|num-b=22|num-a=24}}
 +
{{AMC12 box|year=2020|ab=A|num-b=19|num-a=21}}
 +
 +
[[Category: Introductory Geometry Problems]]
 
{{MAA Notice}}
 
{{MAA Notice}}

Revision as of 11:57, 7 November 2020

The following problem is from both the 2020 AMC 12A #20 and 2020 AMC 10A #23, so both problems redirect to this page.

Problem

Let $T$ be the triangle in the coordinate plane with vertices $(0,0), (4,0),$ and $(0,3).$ Consider the following five isometries (rigid transformations) of the plane: rotations of $90^{\circ}, 180^{\circ},$ and $270^{\circ}$ counterclockwise around the origin, reflection across the $x$-axis, and reflection across the $y$-axis. How many of the $125$ sequences of three of these transformations (not necessarily distinct) will return $T$ to its original position? (For example, a $180^{\circ}$ rotation, followed by a reflection across the $x$-axis, followed by a reflection across the $y$-axis will return $T$ to its original position, but a $90^{\circ}$ rotation, followed by a reflection across the $x$-axis, followed by another reflection across the $x$-axis will not return $T$ to its original position.)

$\textbf{(A) } 12 \qquad \textbf{(B) } 15 \qquad \textbf{(C) } 17 \qquad \textbf{(D) } 20 \qquad \textbf{(E) } 25$

Solution

[asy] size(10cm); Label f;  f.p=fontsize(6);  xaxis(-6,6,Ticks(f, 2.0));  yaxis(-6,6,Ticks(f, 2.0));  filldraw(origin--(4,0)--(0,3)--cycle, gray, black+linewidth(1)); [/asy]

First, any combination of motions we can make must reflect $T$ an even number of times. This is because every time we reflect $T$, it changes orientation. Once $T$ has been flipped once, no combination of rotations will put it back in place because it is the mirror image; however, flipping it again changes it back to the original orientation. Since we are only allowed $3$ transformations and an even number of them must be reflections, we either reflect $T$ $0$ times or $2$ times.


Case 1: 0 reflections on T


In this case, we must use $3$ rotations to return $T$ to its original position. Notice that our set of rotations, $\{90^\circ,180^\circ,270^\circ\}$, contains every multiple of $90^\circ$ except for $0^\circ$. We can start with any two rotations $a,b$ in $\{90^\circ,180^\circ,270^\circ\}$ and there must be exactly one $c \equiv -a - b \pmod{360^\circ}$ such that we can use the three rotations $(a,b,c)$ which ensures that $a + b + c \equiv 0^\circ \pmod{360^\circ}$. That way, the composition of rotations $a,b,c$ yields a full rotation. For example, if $a = b = 90^\circ$, then $c \equiv -90^\circ - 90^\circ = -180^\circ \pmod{360^\circ}$, so $c = 180^\circ$ and the rotations $(90^\circ,90^\circ,180^\circ)$ yields a full rotation.

The only case in which this fails is when $c$ would have to equal $0^\circ$. This happens when $(a,b)$ is already a full rotation, namely, $(a,b) = (90^\circ,270^\circ),(180^\circ,180^\circ),$ or $(270^\circ,90^\circ)$. However, we can simply subtract these three cases from the total. Selecting $(a,b)$ from $\{90^\circ,180^\circ,270^\circ\}$ yields $3 \cdot 3 = 9$ choices, and with $3$ that fail, we are left with $6$ combinations for case 1.


Case 2: 2 reflections on T


In this case, we first eliminate the possibility of having two of the same reflection. Since two reflections across the x-axis maps $T$ back to itself, inserting a rotation before, between, or after these two reflections would change $T$'s final location, meaning that any combination involving two reflections across the x-axis would not map $T$ back to itself. The same applies to two reflections across the y-axis.

Therefore, we must use one reflection about the x-axis, one reflection about the y-axis, and one rotation. Since a reflection about the x-axis changes the sign of the y component, a reflection about the y-axis changes the sign of the x component, and a $180^\circ$ rotation changes both signs, these three transformation composed (in any order) will suffice. It is therefore only a question of arranging the three, giving us $3! = 6$ combinations for case 2.

Combining both cases we get $6 + 6 = \boxed{\textbf{(A) } 12}$

Solution 2(Rewording solution 1)

As in the previous solution, note that we must have either 0 or 2 reflections because of orientation since reflection changes orientation that is impossible to fix by rotation. We also know we can't have the same reflection twice, since that would give a net of no change and would require an identity rotation.

Suppose there are no reflections. Denote $90^{\circ}$ as 1, $180^{\circ}$ as 2, and $270^{\circ}$ as 3, just for simplification purposes. We want a combination of 3 of these that will sum to either 4 or 8(0 and 12 is impossible since the minimum is 3 and the max is 9). 4 can be achieved with any permutation of $(1-1-2)$ and 8 can be achieved with any permutation of $(2-3-3)$. This case can be done in $3+3=6$ ways.

Suppose there are two reflections. As noted already, they must be different, and as a result will take the triangle to the opposite side of the origin if we don't do any rotation. We have 1 rotation left that we can do though, and the only one that will return to the original position is 2, which is $180^{\circ}$ AKA reflection across origin. Therefore, since all 3 transformations are distinct. The three transformations can be applied anywhere since they are commutative(think quadrants). This gives $6$ ways.

$6+6=\boxed{(A)12}$

Video Solution

Education, The Study of Everything

https://youtu.be/SBhkM2frTUA

https://youtu.be/5TjrDCxTm7Q - Richard Rusczyk

Video Solution

https://www.youtube.com/watch?v=iXwvTmFvo0c ~ MathEx

See Also

2020 AMC 10A (ProblemsAnswer KeyResources)
Preceded by
Problem 22
Followed by
Problem 24
1 2 3 4 5 6 7 8 9 10 11 12 13 14 15 16 17 18 19 20 21 22 23 24 25
All AMC 10 Problems and Solutions
2020 AMC 12A (ProblemsAnswer KeyResources)
Preceded by
Problem 19
Followed by
Problem 21
1 2 3 4 5 6 7 8 9 10 11 12 13 14 15 16 17 18 19 20 21 22 23 24 25
All AMC 12 Problems and Solutions

The problems on this page are copyrighted by the Mathematical Association of America's American Mathematics Competitions. AMC logo.png